Exam 2 Conceptual Questions

Pataasin ang iyong marka sa homework at exams ngayon gamit ang Quizwiz!

c

What is the velocity of all electromagnetic waves in a vacuum? a) Electromagnetic waves do not propagate forward; hence, their velocity is zero. b) The velocity of electromagnetic waves depends upon their frequency. c) The velocity of electromagnetic waves is nearly equal to 3 × 108 m/s. d) The velocity of electromagnetic waves depends upon their wavelength. e) The velocity of electromagnetic waves depends upon their amplitude.

b

A charged particle enters into a uniform magnetic field such that its velocity vector is perpendicular to the magnetic field vector. Ignoring the particle's weight, what type of path will the particle follow? a) The charged particle will follow a straight-line path. b) The charged particle will follow a circular path. c) The charged particle will follow a spiral path. d) The charged particle will follow a parabolic path.

a

A circular loop of wire lies flat on a level table top. A bar magnet is held stationary above the circular loop with its north pole point downward. As viewed from above, in what direction does the induced current flow in the loop of wire? a) No current is induced in the loop of wire. b) The direction of the induced current cannot be determined from the given information. c) An induced current flows counterclockwise in the loop of wire. d) An induced current flows clockwise in the loop of wire.

b

A convex lens is placed on a flat glass plate and illuminated from above with monochromatic red light. When viewed from above, concentric bands of red and dark are observed. What does one observe at the exact center of the lens where the lens and the glass plate are in direct contact? a) One observes a bright spot that is some color other than red. b) One observes a dark spot. c) One observes a bright red spot. d) One observes a rainbow of color.

c

A double-slit diffraction pattern is formed on a distant screen. If the separation between the slits decreases, what happens to the distance between interference fringes? Assume the angles involved remain small. a) The distance between interference fringes remains the same. b) The distance between interference fringes also decreases. c) The distance between interference fringes increases. d) The effect cannot be determined unless the distance between the slits and the screen is known.

b

A double-slit experiment yields an interference pattern due to the path length difference from light traveling through one slit versus the other. Why does a single slit show a diffraction pattern? a) The wavelength of the light is shorter than the slit. b) There is a path length difference from waves originating at different parts of the slit. c) The single slit must have something in the middle of it, causing it to act like a double slit. d) The light passing through the slit interferes with light that does not pass through.

b

A long, straight, vertical wire carries a current upward. Due east of this wire, in what direction does the magnetic field point? a) south b) north c) downward d) west e) east f) upward

a,d

A loop rests in the plane of a page of textbook while a magnetic field is directed into the page. A clockwise current is induced Check all that apply. a) when the size of the loop decreases. b) when the loop is moved sideways across the page. c) when the magnetic field gets stronger. d) when the magnetic field is tilted so it is no longer perpendicular to the page.

c

A nonconducting plastic hoop is held in a magnetic field that points out of the figure . As the strength of the field increases, a) an induced emf will be produced that causes a counterclockwise current. b) no induced emf will be produced. c) an induced emf will be produced but no current. d) an induced emf will be produced that causes a clockwise current.

0.71 m

A person whose eyes are H= 1.55 m above the floor stands 2.20 m in front of a vertical plane mirror whose bottom edge is 38 cm above the floor (Figure 1). What is the horizontal distance x to the base of the wall supporting the mirror of the nearest point on the floor that can be seen reflected in the mirror?

a,e

A ray of light crosses a boundary between two transparent materials. The medium the ray enters has a larger optical density. Which of the following statements are true? Check all that apply. a) The wavelength of the light decreases as it enters into the medium with the greater optical density. b) The speed of the light remains constant as it transitions between materials. c) The frequency of the light decreases as it enters into the medium with the greater optical density. d) The speed of the light increases as it enters the medium with the greater optical density. e) The frequency of the light remains constant as it transitions between materials. f) The wavelength of the light remains constant as it transitions between materials.

a

A ray of light passes from air into a block of clear plastic. How does the angle of incidence in the air compare to the angle of refraction in the plastic? a) The angle of incidence is greater than the angle of refraction. b) The angle of incidence is equal to the angle of refraction. c) The angle of incidence is less than the angle of refraction. d) The two angles cannot be compared without additional information.

a

A single-slit diffraction pattern is formed on a distant screen. If the distance from the slit to the screen is doubled, by what factor will the width of the central bright fringe on the screen change? Assume the angles involved remain small. a) The width of the central bright fringe will be twice its original size. b) The width of the central bright fringe will remain the same. c) The width of the central bright fringe will be reduced to one-half of its original size. d) The width of the central bright fringe will be four times its original size. e) The width of the central bright fringe will be eight times its original size. f) The width of the central bright fringe will be reduced to one-quarter of its original size.

a

A single-slit diffraction pattern is formed on a distant screen. If the width of the single slit through which light passes is reduced, what happens to the width of the central bright fringe? Assume the angles involved remain small. a) The central bright fringe becomes wider. b) The central bright fringe becomes narrower. c) The effect cannot be determined unless the distance between the slit and the screen is known. d) The central bright fringe remains the same size.

b

A square loop moves to the right from an area where B=0, completely through a region containing a uniform magnetic field directed into the figure (Figure 1), and then out to B=0 after point L A current is induced in the loop a) only as it passes line J b) as it passes line J or line L c) only as it passes line K d) only as it passes line L e) as it passes all three lines.

a

An electron is traveling horizontally toward the north in a uniform magnetic field that is directed vertically downward. In what direction does the magnetic force act on the electron? a) east b) west c) south d) north e) upward f) downward

a,b,d,e

Consider a magnetic force acting on an electric charge in a uniform magnetic field. Which of the following statements are true? Check all that apply. a) The direction of the magnetic force acting on a moving charge in a magnetic field is perpendicular to the direction of the magnetic field. b) A magnetic force is exerted on an electric charge moving through a uniform magnetic field. c) A magnetic force is exerted on a stationary electric charge in a uniform magnetic field. d) An electric charge moving perpendicular to a magnetic field experiences a magnetic force. e) The direction of the magnetic force acting on a moving electric charge in a magnetic field is perpendicular to the direction of motion. f) An electric charge moving parallel to a magnetic field experiences a magnetic force.

b

Consider an electromagnetic wave propagating through a region of empty space. How is the energy density of the wave partitioned between the electric and magnetic fields? a) The energy density of an electromagnetic wave is 25% in the electric field and 75% in the magnetic field. b) The energy density of an electromagnetic wave is equally divided between the magnetic and electric fields. c) The energy density of an electromagnetic wave is 25% in the magnetic field and 75% in the electric field. d) The energy density of an electromagnetic wave is entirely in the magnetic field. e) The energy density of an electromagnetic wave is entirely in the electric field.

a`

For a charged particle, a constant magnetic field can be used to change a) only the direction of the particle's velocity. b) only the magnitude of the particle's velocity. c) both the magnitude and direction of the particle's velocity. d) eNone of the above.

c

If the Earth-Sun distance were doubled, the intensity of radiation from the Sun that reaches the Earth's surface would a) quadruple b) double c) drop to 1/4 d) drop to 1/2

d

If the distance from your eye's lens to the retina is shorter than for a normal eye, you will struggle to see objects that are a) colorful. b) far away. c) moving fast. d) nearby.

b

In a double-slit experiment, two beams of coherent light traveling different paths arrive on a screen some distance away. What is the path difference between the two waves corresponding to the third bright band out from the central bright band? a) The path difference between the two waves is one-half of a wavelength. b) The path difference between the two waves is three wavelengths. c) The path difference between the two waves is one and one-half wavelengths. d) The path difference between the two waves is one wavelength. e) The path difference between the two waves is four wavelengths. f) The path difference between the two waves is two wavelengths.

d

In empty space, which quantity is always larger for X-ray radiation than for a radio wave? a) Amplitude. b) Wavelength. c) Speed. d) Frequency.

a,b,c,e

In the figure, a conducting rod is shown moving in a uniform magnetic field . Which of the following statements are true? Check all that apply a) The magnitude of the potential difference in the rod is proportional to the velocity v of the rod. b) The magnitude of the potential difference in the rod is proportional to the strength of the magnetic field. c) The free charges in the rod are acted upon by a magnetic force. d) The magnitude of the potential difference in the rod is inversely proportional to the length L of the rod. e) An electric field is established in the rod directed from point a to point b.

d

Light passing through a double-slit arrangement is viewed on a distant screen. The interference pattern observed on the screen would have the widest spaced fringes for the case of a) blue light and a large slit spacing. b) red light and a large slit spacing. c) blue light and a small slit spacing. d) red light and a small slit spacing.

b,d

Suppose a ray of light traveling in a material with an index of refraction na reaches an interface with a material having an index of refraction nb. Which of the following statements must be true for total internal reflection to occur? Check all that apply. a) na = nb b) The angle of incidence must be greater than the critical angle. c) The angle of incidence must be equal to the critical angle. d) na > nb e) The angle of incidence must be less than the critical angle.

d

Suppose that an electromagnetic wave is traveling toward the east. At one instant at a given point, the electric field vector points upward. What is the direction of the magnetic field at this same given point and instant in time? a) west b) north c) downward d) south e) east f) upward

6.6m

Suppose that you want to take a photograph of yourself as you look at your image in a mirror 3.3 m away. For what distance should the camera lens be focused?

b

The colors in a rainbow are caused by a) different amounts of absorption for light of different colors by the water in the raindrops. b) different amounts of refraction for light of different colors by the water in the raindrops. c) the interaction of the light reflected from different raindrops. d) the downward motion of the raindrops.

e

The magnitude of the magnetic field at a certain distance from a long, straight conductor is represented by B. What is the magnitude of the magnetic field at twice the distance from the conductor? a) At twice the distance, the magnitude of the field is B/4. b) At twice the distance, the magnitude of the field is 2B. c) At twice the distance, the magnitude of the field remains equal to B. d) At twice the distance, the magnitude of the field is 4B. e) At twice the distance, the magnitude of the field is B/2.

c

Two beams of coherent light travel different paths, arriving at point P. If the maximum destructive interference is to occur at point P, what should be the path difference between the two waves? a) The path difference between the two waves should be one wavelength. b) The path difference between the two waves should be one-quarter of a wavelength. c) The path difference between the two waves should be one-half of a wavelength. d) The path difference between the two waves should be two wavelengths. e) The path difference between the two waves should be four wavelengths. f) The path difference between the two waves should be one and one-quarter of a wavelengths.

a,d

Two long parallel wires are placed side by side on a horizontal table. The wires carry equal currents in the same direction. Which of the following statements are true? Check all that apply. a) The magnetic force between the two wires is attractive. b) The magnetic field is a maximum at a point midway between the two wires. c) The magnetic force between the two wires is repulsive. d) The magnetic field at a point midway between the two wires is zero.

c

Two separate but nearby coils are mounted along the same axis. A power supply controls the flow of current in the first coil, and thus the magnetic field it produces. The second coil is connected only to an ammeter. The ammeter will indicate that a current is flowing in the second coil a) only when a steady current flows in the first coil. b) only if the second coil is connected to the power supply by rewiring it to be in series with the first coil. c) only when the current in the first coil changes. d) whenever a current flows in the first coil.

c

What must you do to keep it in focus? a) Decrease the f-stop value. b) Move the lens away from the sensor or film. c) Move the lens closer to the sensor or film. d) Increase the f-stop value. e) None of the above.

a,b,c,d,f

Which of the following are found within the electromagnetic spectrum? Check all that apply. a) ultraviolet radiation b) gamma rays c) X rays d) visible light e) sound waves f) microwave radiation

d

Which of the following can a transformer accomplish? a) Changing power. b) Changing current but not voltage. c) Changing voltage but not current. d) Changing both current and voltage.

e

Which of the following statements about the force on a charged particle due to a magnetic field are not valid? Check all that apply. a) It depends on the particle's velocity. b) It acts at right angles to the direction of the particle's motion. c) It depends on the particle's charge. d) It depends on the strength of the external magnetic field. e) None of the above; all of these statements are valid.

a,d

Which of the following statements are true concerning compound microscopes? Check all that apply. a) In a compound microscope, the image formed by the objective lens is a real image. b) In a compound microscope, the final image is formed by the objective lens. c) The focal length of the objective in a microscope is very large compared to the focal length of the eyepiece. d) In a compound microscope, the final image is a virtual image. e) In a compound microscope, the image formed by the objective lens is smaller than the object.

a,c,d

Which of the following statements are true concerning electromagnetic radiation fields? Check all that apply. a) The magnetic field at any point is perpendicular to the direction of wave travel. b) The electric and magnetic fields at any point are parallel to each other. c) The electric field at any point is perpendicular to the direction of wave travel. d) The electric and magnetic fields are in phase with each other as they propagate through space. e) The electric and magnetic fields are phase-shifted 180° with respect to each other as they propagate through space.

c,d

Which of the following statements are true concerning electromagnetic waves in a vacuum? Check all that apply. a) Electromagnetic waves traveling in a vacuum are longitudinal waves. b) Electromagnetic waves can travel in a vacuum only because a medium prohibits the forward propagation of the electric and magnetic fields. c) Electromagnetic waves travel in a vacuum with a definite and unchanging speed. d) When electromagnetic waves travel in vacuum, there is a definite, constant ratio between the magnitudes of the electric and magnetic fields.

b,d,e

Which of the following statements are true concerning the creation of magnetic fields? Check all that apply. a) A distribution of electric charges at rest creates a magnetic field at all points in the surrounding region. b) A moving electric charge creates a magnetic field at all points in the surrounding region. c) A single stationary electric charge creates a magnetic field at all points in the surrounding region. d) An electric current in a conductor creates a magnetic field at all points in the surrounding region. e) A permanent magnet creates a magnetic field at all points in the surrounding region.

a,b,d

Which of the following statements are true concerning the eye? Check all that apply. a) Nearsightedness can be corrected with a diverging lens. b) Astigmatism refers to a defect in which the surface of the cornea is not spherical. Instead, it is more sharply curved in one plane than another. c) Farsightedness can be corrected with a diverging lens. d) The far point of the normal eye is infinity. e) The hyperopic eye focuses rays from an object at infinity in front of the retina; the myopic eye focuses rays from an object at infinity behind the retina.

a,d

Which of the following statements are true concerning the reflection of light? Check all that apply. a) The reflection of light from a rough surface is called diffuse reflection. b) For diffuse reflection, the angle of incidence is greater than the angle of reflection. c) The angle of incidence is equal to the angle of reflection only when a ray of light strikes a plane mirror. d) The reflection of light from a smooth surface is called specular reflection. e) For specular reflection, the angle of incidence is less than the angle of reflection.

b,c,f

Which of the following statements are true for an image formed by a plane mirror? Check all that apply. a) The image formed by a plane mirror is always a real image. b) The image formed by a plane mirror is always an upright image. c) The image always appears to be located the same distance behind the mirror as the object is located in front of the mirror. d) The image is always larger than the object. e) The image formed by a plane mirror is always an inverted image. f) The image formed by a plane mirror is always a virtual image.

b,e

Which of the following statements are true for images formed by spherical mirrors? Check all that apply. a) A concave mirror always produces an image that is the same size as the object. b) A convex mirror always produces a virtual image of an object placed in front of it. c) A convex mirror always produces an image that is the same size as the object. d) A concave mirror always produces an inverted image on an object placed in front of it. e) A convex mirror always produces an upright image of an object placed in front of it. f) A concave mirror always produces a real image of an object placed in front of it.

d,f

Which of the following statements are true for images formed by thin lenses? Check all that apply. a) A converging lens always produces an image that is larger than the object. b) A converging lens always produces a real image of an object placed in front of it. c) A diverging lens always produces an image that is larger than the object. d) A diverging lens always produces a virtual image of an object placed in front of it. e) A converging lens always produces an inverted image of an object placed in front of it. f) A diverging lens always produces an upright image of an object placed in front of it.

a,c,e

Which of the following statements are true for magnetic field lines? Check all that apply. a) Magnetic field lines can never intersect. b) Magnetic field lines are close together in regions of space where the magnitude of the magnetic field is weak, and they are father apart in regions where it is strong. c) At every point in space, the magnetic field vector at that point is tangent to the magnetic field line through that point. d) Magnetic field lines point in the direction of the magnetic force acting on a charge. e) Unlike electric field lines, magnetic field lines are continuous.

a,b,c

Which of the following statements are true for magnetic force acting on a current-carrying wire in a uniform magnetic field? Check all that apply. a) The direction of the magnetic force acting on a current-carrying wire in a uniform magnetic field is perpendicular to the direction of the current. b) The magnetic force on the current-carrying wire is strongest when the current is perpendicular to the magnetic field lines. c) The direction of the magnetic force acting on a current-carrying wire in a uniform magnetic field is perpendicular to the direction of the field. d) The magnetic force on the current-carrying wire is strongest when the current is parallel to the magnetic field lines.

d,e

Which of the following statements are true regarding electric generators? Check all that apply. a) An electric generator is used to increase or decrease an alternating current voltage. b) An electric generator converts electrical energy into magnetic energy. c) An electric generator creates a time-independent induced voltage. d) An electric generator creates a time-dependent induced voltage. e) An electric generator converts mechanical energy into electrical energy.

e,f

Which of the following statements are true regarding transformers? Check all that apply. a) A transformer is used to increase or decrease a direct current voltage. b) A transformer converts mechanical energy into electrical energy. c) In a transformer, the power input is less than the power output. d) In a transformer, if the primary coil contains more loops than the secondary coil, then it is a step-up transformer. e) A transformer is used to increase or decrease an alternating current voltage. f) In a transformer, if the secondary coil contains more loops than the primary coil, then it is a step-up transformer.

a,c

Which of the following statements are true? Check all that apply. a) A time-varying electric field will produce a magnetic field. b) Time-varying electric and magnetic fields can propagate through space only if there is no matter in their path. c) A time-varying magnetic field will produce an electric field. d) Electric and magnetic fields can be treated independently only if they vary in time.


Kaugnay na mga set ng pag-aaral

Pharm Ch. 15 Antiparkinson Drugs

View Set

Chapter 43: Assessment and Management of Patients with Hepatic Disorders

View Set

Section 4.2 Part 1: Simplifying and Multiplying Fractions

View Set

Lecture 18: Gantt Charts in Project Management

View Set